Difference between revisions of "2004 AIME II Problems/Problem 9"
I like pie (talk | contribs) m |
(temporary saving) |
||
Line 1: | Line 1: | ||
== Problem == | == Problem == | ||
− | A sequence of positive integers with <math> a_1=1 </math> and <math> a_9+a_{10}=646 </math> is formed so that the first three terms are in geometric progression, the second, third, and fourth terms are in arithmetic progression, and, in general, for all <math> n\ge1, </math> the terms <math> a_{2n-1}, a_{2n}, a_{2n+1} </math> are in geometric progression, and the terms <math> a_{2n}, a_{2n+1}, </math> and <math> a_{2n+2} </math> are in arithmetic progression. Let <math> a_n </math> be the greatest term in this sequence that is less than 1000. Find <math> n+a_n. </math> | + | A [[sequence]] of positive integers with <math> a_1=1 </math> and <math> a_9+a_{10}=646 </math> is formed so that the first three terms are in [[geometric sequence|geometric progression]], the second, third, and fourth terms are in [[arithmetic sequence|arithmetic progression]], and, in general, for all <math> n\ge1, </math> the terms <math> a_{2n-1}, a_{2n}, a_{2n+1} </math> are in geometric progression, and the terms <math> a_{2n}, a_{2n+1}, </math> and <math> a_{2n+2} </math> are in arithmetic progression. Let <math> a_n </math> be the greatest term in this sequence that is less than <math>1000</math>. Find <math> n+a_n. </math> |
== Solution == | == Solution == | ||
− | {{ | + | Let <math>x = a_2</math>; then solving for the next several terms, we find that <math>a_3 = x^2,\ a_4 = x(2x-1),\ a_5 = (2x-1)^2,\ a_6 = (2x-1)(3x-2)</math>, and in general, <math>a_{2n} = [(n-1)x - (n-2)][nx - (n-1)],</math> <math> a_{2n+1} = [nx -(n-1)]^2</math>. This we can easily show by [[induction]]: since <math>a_{2n} = 2a_{2n-1} - a_{2n-2} = 2[(n-1)x-(n-2)]^2 - [(n-2)x-(n-3)][(n-1)x-(n-2)] = []</math>. The answer is <math>957 + 16</math>. |
== See also == | == See also == | ||
{{AIME box|year=2004|n=II|num-b=8|num-a=10}} | {{AIME box|year=2004|n=II|num-b=8|num-a=10}} | ||
+ | |||
+ | [[Category:Intermediate Algebra Problems]] |
Revision as of 17:13, 8 June 2008
Problem
A sequence of positive integers with and is formed so that the first three terms are in geometric progression, the second, third, and fourth terms are in arithmetic progression, and, in general, for all the terms are in geometric progression, and the terms and are in arithmetic progression. Let be the greatest term in this sequence that is less than . Find
Solution
Let ; then solving for the next several terms, we find that , and in general, . This we can easily show by induction: since . The answer is .
See also
2004 AIME II (Problems • Answer Key • Resources) | ||
Preceded by Problem 8 |
Followed by Problem 10 | |
1 • 2 • 3 • 4 • 5 • 6 • 7 • 8 • 9 • 10 • 11 • 12 • 13 • 14 • 15 | ||
All AIME Problems and Solutions |